Twice a square

Find a positive integer N N such that p 3 + N 2 p^3 + N^2 is a perfect square for 2 distinct (positive) primes p p .

Details and assumptions

You may use a theorem of Ljunggren, which states that the equation x 2 + x + 1 = y 3 x^2+x+1=y^3 has only the following integer solutions: ( x , y ) = ( 0 , 1 ) , ( 1 , 1 ) , ( 18 , 7 ) , ( 19 , 7 ) (x,y)=(0,1), (-1,1), (18,7), (-19,7) .


The answer is 171.

This section requires Javascript.
You are seeing this because something didn't load right. We suggest you, (a) try refreshing the page, (b) enabling javascript if it is disabled on your browser and, finally, (c) loading the non-javascript version of this page . We're sorry about the hassle.

13 solutions

Andrei Zanfir
Jul 29, 2013

We have p 3 = k 2 N 2 = ( k N ) ( k + N ) p^{3} = k^{2} - N^{2} = (k-N)(k+N)

Because p p is prime we can have the cases:

  1. k N = p k-N = p

    k + N = p 2 k+N = p^2

  2. k N = 1 k-N = 1

    k + N = p 3 k+N = p^3

We know that they are two distinct solutions, but each case yields exactly one p p , given N N . So both cases must apply. Therefore:

From 1: N = p 1 2 p 1 2 N = \frac{p_{1}^{2} - p_{1}}{2}

From 2: N = p 2 3 1 2 N = \frac{p_{2}^{3} - 1}{2}

So, we must find p 1 p_{1} and p 2 p_{2} prime numbers, such that:

p 2 3 = p 1 2 p 1 + 1 p_{2}^{3} = p_{1}^{2} - p_{1} + 1

Using the theorem handed, we find that p 1 = 19 p_{1} = 19 and p 2 = 7 p_{2} = 7 . Therefore N = 171 N=171 is the solution.

Moderator note:

The theorem of Ljunggren is a particular case of finding all integer points on elliptic curves, and is a non-trivial computational problem.

Jimmy Kariznov
Jul 29, 2013

Suppose p 3 + N 2 = a 2 p^3 + N^2 = a^2 for some positive integer a a . (Clearly, a > N a > N .)

Then, we have p 3 = a 2 N 2 = ( a N ) ( a + N ) p^3 = a^2 - N^2 = (a-N)(a+N) . (both factors are positive)

Since the factors of p 3 p^3 are 1 , p , p 2 , p 3 1,p,p^2,p^3 and a N < a + N a-N < a+N , we have either:

Case 1: a N = p a-N = p and a + N = p 2 a+N = p^2 , which yields N = p 2 p 2 N = \dfrac{p^2-p}{2}

Case 2: a N = 1 a-N = 1 and a + N = p 3 a+N = p^3 , which yields N = p 3 1 2 N = \dfrac{p^3-1}{2}

If p 3 + N 2 p^3+N^2 is a perfect square for two distinct primes p = p 1 , p 2 p = p_1,p_2 , then:

N = p 1 2 p 1 2 = p 2 3 1 2 p 2 3 = p 1 2 p 1 + 1 = ( p 1 1 ) 2 + ( p 1 1 ) + 1 N = \dfrac{p^2_1-p_1}{2} = \dfrac{p^3_2 - 1}{2} \leadsto p^3_2 = p^2_1 - p_1 + 1 = (p_1 -1)^2 + (p_1 - 1) + 1

By Ljunggren's Theorem for x = p 1 1 x = p_1 - 1 and y = p 2 y = p_2 , we must have:

( p 1 1 , p 2 ) { ( 0 , 1 ) , ( 1 , 1 ) , ( 18 , 7 ) , ( 19 , 7 ) } (p_1 - 1,p_2) \in \{(0,1), (-1,1), (18,7), (-19,7)\}

i.e. ( p 1 , p 2 ) { ( 1 , 1 ) , ( 0 , 1 ) , ( 19 , 7 ) , ( 18 , 7 ) } (p_1,p_2) \in \{(1,1), (0,1), (19,7), (-18,7)\} .

The only solution with both ( p 1 , p 2 ) (p_1,p_2) prime is ( 19 , 7 ) (19,7) .

This gives N = 1 9 2 19 2 = 7 3 1 2 = 171 N = \dfrac{19^2 -19}{2} = \dfrac{7^3 - 1}{2} = \boxed{171} .

Moderator note:

Another direct substitution which you can use is x = p 1 x = -p_1 and y = p 2 y = p_2 , which makes it easier to identify the pair of primes.

Ivan Koswara
Jul 30, 2013

Let p 1 3 + N 2 = x 1 2 p_1^3 + N^2 = x_1^2 and p 2 3 + N 2 = x 2 2 p_2^3 + N^2 = x_2^2 for some positive integers x 1 , x 2 x_1, x_2 , where p 1 , p 2 p_1, p_2 are primes.

Now, x 1 2 N 2 = p 1 3 x_1^2 - N^2 = p_1^3 , so p 1 3 = ( x 1 + N ) ( x 1 N ) p_1^3 = (x_1 + N)(x_1 - N) . Since p 1 3 p_1^3 only has four factors ( 1 , p 1 , p 1 2 , p 1 3 1, p_1, p_1^2, p_1^3 ), and x 1 + N > x 1 N x_1 + N > x_1 - N , we only have two possible solutions: ( x 1 + N , x 1 N ) = ( p 1 2 , p 1 ) , ( p 1 3 , 1 ) (x_1 + N, x_1 - N) = (p_1^2, p_1), (p_1^3, 1) . We can solve each solution for N N (add the two equations and divide by 2 2 ): N = p 1 2 p 1 2 , p 1 3 1 2 N = \dfrac{p_1^2 - p_1}{2}, \dfrac{p_1^3 - 1}{2} . A similar solution is obtained for p 2 p_2 .

Suppose that both solutions are in the same form: N = p 1 2 p 1 2 N = \dfrac{p_1^2 - p_1}{2} and N = p 2 2 p 2 2 N = \dfrac{p_2^2 - p_2}{2} , or N = p 1 3 1 2 N = \dfrac{p_1^3 - 1}{2} and N = p 2 3 1 2 N = \dfrac{p_2^3 - 1}{2} .

  • Case 1: N = p 1 2 p 1 2 N = \dfrac{p_1^2 - p_1}{2} and N = p 2 2 p 2 2 N = \dfrac{p_2^2 - p_2}{2} .

Let f ( a ) = a 2 a 2 N f(a) = a^2 - a - 2N . Then p 1 , p 2 p_1, p_2 are roots of f f . Suppose they are different solutions, then p 1 , p 2 p_1, p_2 are all the roots of f f . Then by Vieta's formulas , p 1 p 2 = 2 N p_1p_2 = -2N . Since N > 0 N > 0 , 2 N < 0 -2N < 0 , so one of p 1 , p 2 p_1, p_2 is negative, contradicting the assumption that they are primes.

  • Case 2: N = p 1 3 1 2 N = \dfrac{p_1^3 - 1}{2} and N = p 2 3 1 2 N = \dfrac{p_2^3 - 1}{2} .

Let a 3 = 2 N 1 a^3 = 2N - 1 . Since a a is real, there is exactly one solution of a a . But a = p 1 , a = p 2 a = p_1, a = p_2 are both solutions, so p 1 = p 2 p_1 = p_2 , contradicting our assumption that they are distinct.

So they are of different forms. WLOG assume that N = p 1 2 p 1 2 N = \dfrac{p_1^2 - p_1}{2} and N = p 2 3 1 2 N = \dfrac{p_2^3 - 1}{2} . Equate the two equations:

p 1 2 p 1 2 = p 2 3 1 2 \dfrac{p_1^2 - p_1}{2} = \dfrac{p_2^3 - 1}{2}

p 1 2 p 1 = p 2 3 1 p_1^2 - p_1 = p_2^3 - 1

p 1 2 p 1 + 1 = p 2 3 p_1^2 - p_1 + 1 = p_2^3

Let x = p 1 , y = p 2 x = -p_1, y = p_2 , then our equation becomes x 2 + x + 1 = y 3 x^2 + x + 1 = y^3 . The problem's notes (about a theorem of Ljunggren) gives all solutions ( x , y ) = ( 0 , 1 ) , ( 1 , 1 ) , ( 18 , 7 ) , ( 19 , 7 ) (x,y) = (0,1), (-1,1), (18,7), (-19,7) . Since p 1 2 p_1 \ge 2 , x = p 1 2 x = -p_1 \le -2 , so only one solution fits: ( x , y ) = ( 19 , 7 ) (x,y) = (-19,7) . This gives p 1 = 19 , p 2 = 7 p_1 = 19, p_2 = 7 , and so N = 1 9 2 19 2 = 7 3 1 2 = 171 N = \dfrac{19^2 - 19}{2} = \dfrac{7^3 - 1}{2} = \boxed{171} . This can be verified to work (when p = 19 p = 19 , p 3 + N 2 = 1 9 3 + 17 1 2 = 19 0 2 p^3 + N^2 = 19^3 + 171^2 = 190^2 , and when p = 7 p = 7 , p 3 + N 2 = 7 3 + 17 1 2 = 17 2 2 p^3 + N^2 = 7^3 + 171^2 = 172^2 ).

Aditya Parson
Jul 30, 2013

Let p 3 + N 2 = x 2 p^3 + N^2 = x^2 where x x is a positive integer.

p 3 = x 2 N 2 \Rightarrow p^3=x^2 - N^2

p 3 = ( x + N ) ( x n ) p^3 = (x+N)(x-n)

Now we have that either ( ( x + N ) , ( x N ) ) ( ( p 3 , 1 ) , ( p 2 , p ) ) ((x+N), (x-N)) \in ({(p^3,1), (p^2, p) }) [Here, we use the clear fact that x + N > x N x+N > x-N since both x , N x,N are positive integers].

Solving for N N gives us that 2 N = p 3 1 , p 2 p 2N=p^3 - 1 , p^2 - p

Let the other prime be q q and q p q \neq p and we get that 2 N = q 3 1 , q 2 q 2N=q^3-1, q^2- q .

Suppose when 2 N = p 3 1 2N=p^3 -1 let us also have that 2 N = q 3 1 2N= q^3-1 . This clearly implies that q = p q = p which is not true, according to our previous assumption.

Now, when 2 N = p 2 p = q 2 q 2N= p^2 -p= q^2- q :

( p q ) ( p + q 1 ) = 0 \Rightarrow (p-q)(p+q-1)=0

Now, since p q p + q = 1 p \neq q \Rightarrow p+q=1 . This is not possible since p , q p,q are positive integers and that reason itself is enough to reject this case as well.

.

Let us now suppose that 2 N = p 3 1 = q 2 q 2N= p^3-1= q^2 - q

We get that p 3 = q 2 q + 1 p^3= q^2 - q +1 . This is awesome since if f ( z ) = z 2 + z + 1 f(z)=z^2 + z +1 then f ( z ) = z 2 z + 1 f(-z)= z^2 -z +1 . As such the various solutions to the equation, according to Ljunggren's theorem, are : ( 0 , 1 ) , ( 1 , 1 ) , ( 18 , 7 ) , ( 19 , 7 ) (0,1), (1,1) , (-18, 7), (19,7) .

Clearly we are only interested in ( q , p ) = ( 19 , 7 ) (q,p)= (19,7) .

Substituting for p = 7 p=7 we get 2 N = 7 3 1 2N=7^3 -1

N = 171 \Rightarrow N= 171 .

We get the same value for N N if we let 2 N = q 3 1 = p 2 p 2N= q^3-1= p^2 - p .

So our answer is 171 \boxed{171} .

Thomas Beuman
Aug 1, 2013

Let p p and q q be the two distinct primes, with p < q p<q , and a a and b b be the roots of the perfect sqaures, i.e.

a 2 = p 3 + N 2 and b 2 = q 3 + N 2 . a^2 = p^3+N^2 \quad \text{and} \quad b^2 = q^3+N^2.

After rearranging:

p 3 = a 2 N 2 = ( a N ) ( a + N ) and q 3 = ( b N ) ( b + N ) . p^3 = a^2-N^2 = (a-N)(a+N) \quad \text{and} \quad q^3 = (b-N)(b+N).

Now p 3 p^3 has only two (positive) factorizations: p 3 = 1 p 3 p^3 = 1 \cdot p^3 and p 3 = p p 2 p^3 = p \cdot p^2 . The same is of course true for q 3 q^3 . From the condition p < q p<q it follows that

a N = 1 and a + N = p 3 , b N = q and b + N = q 2 . a-N = 1 \quad \text{and} \quad a+N = p^3, \\ b-N = q \quad \text{and} \quad b+N = q^2.

Some further manipulation:

N = 1 2 ( ( a + N ) ( a N ) ) = 1 2 ( p 3 1 ) = 1 2 ( ( b + N ) ( b N ) ) = 1 2 ( q 2 q ) , N = \frac12((a+N)-(a-N)) = \frac12(p^3-1) \\ \quad = \frac12((b+N)-(b-N)) = \frac12(q^2-q),

and thus

p 3 = q 2 q + 1 = ( q ) 2 + ( q ) + 1. p^3 = q^2-q+1 = (-q)^2 + (-q) + 1.

Now we can invoke the theorem of Ljunggren, to determine that the only solution is p = 7 p=7 and q = 19 -q=-19 , with

N = 1 2 ( 7 3 1 ) = 1 2 ( 1 9 2 19 ) = 171 N = \frac12(7^3-1) = \frac12(19^2-19) = \boxed{171}

The squares are 7 3 + 17 1 2 = 17 2 2 7^3+171^2 = 172^2 and 1 9 3 + 17 1 2 = 19 0 2 19^3+171^2 = 190^2 .

David Vaccaro
Aug 1, 2013

Rearranging n 2 + p 3 = m 2 n^{2}+p^{3}=m^{2} gives ( m n ) ( n + m ) = p 3 (m-n)(n+m)=p^{3} . Since p p is prime there are only two possible factorisations of p 3 p^{3} and our two solutions are m 1 n = 1 , n + m 1 = p 1 3 m_{1}-n=1 , n+m_{1}=p_{1}^{3} and m 2 n = p 2 , n + m 2 = p 2 2 m_{2}-n=p_{2}, n+m_{2}=p_{2}^{2}

Solving each pair of equations gives:

  • 2 n = p 1 3 1 = p 2 2 p 2 2n=p_{1}^{3}-1=p_{2}^{2}-p_{2}

Therefore x = p 1 x=-p_{1} and y = p 2 y=p_{2} are solutions of y 3 = x 2 x + 1 y^{3}=x^{2}-x+1 and by the hint the only prime solutions are p 1 = 19 , p 2 = 7 p_{1}=19, p_{2}=7 . Substituting back in our expression for 2 n 2n we get 2 n = 7 3 1 = 342 2n=7^{3}-1=342 and n = 171 n=171 .

We are given that N 2 + p 3 = m 2 N^2 + p^3 = m^2 for some positive integer m m . Let m = N + k m = N + k . Then N 2 + p 3 = ( N + k ) 2 = N 2 + 2 N k + k 2 N^2 + p^3 = (N + k)^2 = N^2 + 2Nk + k^2 \to p 3 = 2 N k + k 2 = k ( 2 N + k ) p^3 = 2Nk + k^2 = k(2N + k) . This gives us that k , ( 2 N + k ) p 3 k,(2N+k) \mid p^3 , and since 2 N + k > k 2N + k > k and p p is prime, k k must be 1 1 or p p .

Let the two distinct primes that satisfy the equation be p 1 , p 2 p_1,p_2 . Then k 1 ( 2 N + k 1 ) = p 1 3 k_1(2N + k_1) = p_1^3 and k 2 ( 2 N + k 2 ) = p 2 3 k_2(2N + k_2) = p_2^3 . Without loss of generality, let k 1 = 1 , k 2 = p 2 k_1 = 1, k_2 = p_2 . Then we have the following system of equations:

{ 2 N + 1 = p 1 3 2 N + p 2 = p 2 2 \begin{cases} \\ 2N + 1 = p_1^3 \\ 2N + p_2 = p_2^2 \end{cases} .

Subtracting the first equation from the second gives us 1 p 2 = p 1 3 p 2 2 1 - p_2 = p_1^3 - p_2^2 \to ( p 2 ) 2 + ( p 2 ) + 1 = p 1 3 (-p_2)^2 + (-p_2) + 1 = p_1^3 . We are given that 19 , 7 -19,7 satisfies the equation ( 19 ) 2 + ( 19 ) + 1 = 7 3 (-19)^2 + (-19) + 1 = 7^3 , and so we let p 2 = 19 , p 1 = 7 p_2 = 19, p_1 = 7 , which gives us that N = 171 N = \fbox{171} .

Tan Likai
Jul 29, 2013

Let p 3 + N 2 = b 2 p^3 + N^2 = b^2 . Then p 3 = ( N b ) ( N + b ) p^3 = (N - b)(N + b) . Since p 3 p^3 can only be splitted in two ways: p p 2 p \cdot p^2 and 1 p 3 1 \cdot p^3 , we split cases:

Case 1: b + N = p 3 , b N = 1 b + N = p^3, b - N = 1

We have b = 1 + p 3 2 = 1 + N b = \frac{1 + p^3}{2} = 1 + N . So we have 1 + p 3 = 2 + 2 N 1 + p^3 = 2 + 2N , which means that N = p 3 1 2 N = \frac{p^3 - 1}{2} .

Case 2: b + N = p 2 , b N = p b + N = p^2, b - N = p

We have b = p 2 + p 2 = p + N b = \frac{p^2 + p}{2} = p + N . So p 2 + p = 2 p + 2 N p^2 + p = 2p + 2N , which means that N = p 2 p 2 N = \frac{p^2 - p}{2} .

To have 2 distinct primes p satisfying the equation, we must have a solution from each case. Let the solution from case 1 be y and the solution from case 2 be x. Then N = y 3 1 2 = x 2 x 2 N = \frac{y^3 - 1}{2} = \frac{x^2 - x}{2} , which means that y 3 = x 2 x + 1 = ( x 1 ) 2 + ( x 1 ) + 1 y^3 = x^2 - x + 1 = (x - 1)^2 + (x - 1) + 1 . By the theorem by Ljunggren, ( x , y ) = ( 1 , 1 ) , ( 0 , 1 ) , ( 19 , 7 ) , ( 18 , 7 ) (x, y) = (1, 1), (0, 1), (19, 7), (-18, 7) , in which only ( 19 , 7 ) (19, 7) satisfies the condition of both being primes. Hence N = 7 3 1 2 = 171 N = \frac{7^3 - 1}{2} = 171 .

Lucas Reis
Jul 28, 2013

We have x 2 + x + 1 = ( P + 1 ) 2 P 2 x^2+x+1=(P+1)^2-P^2 where P = x ( x + 1 ) 2 P=\frac{x(x+1)}{2} .

If ( ) p 3 + N 2 = Q 2 p 3 = Q 2 N 2 (*) p^3+N^2=Q^2\Rightarrow p^3=Q^2-N^2 .

Let be Q = P + 1 , N = P Q=P+1, N=P .

By the Ljunggren's Theorem, the equation ( ) (*) have solution p = 7 p=7 , and 7 7 is actually a prime number.

So x = 18 x=18 it's a solution for ( ) (*) and then N = P = 18 19 2 = 9 19 = 171 N=P=\frac{18*19}{2}=9*19=171 is a solution.

Daniel Chiu
Jul 28, 2013

We have that p 3 + N 2 = k 2 p^3+N^2=k^2 . Factorizing, ( k N ) ( k + N ) = p 3 (k-N)(k+N)=p^3 p 3 = 1 p 3 = p p 2 p^3=1\cdot p^3=p\cdot p^2 . Now, we have two cases: k = N + 1 = p 3 N k=N+1=p^3-N 2 N = p 3 1 2N=p^3-1 or k = N + p = p 2 N k=N+p=p^2-N 2 N = p 2 p 2N=p^2-p Notice that for any N N , each of these gives one possibility for p p , and so if two primes exist, both must work. Therefore, letting the primes be p p and q q , we have 2 N = p 3 1 = q 2 q 2N=p^3-1=q^2-q p 3 = q 2 q + 1 p^3=q^2-q+1 p 3 = ( q ) 2 + ( q ) + 1 p^3=(-q)^2+(-q)+1 By the theorem of Ljunggren, our solutions for ( q , p ) (-q,p) are ( 0 , 1 ) , ( 1 , 1 ) ( 18 , 7 ) ( 19 , 7 ) (0,1),\ (-1,1)\ (18,7)\ (-19,7) Therefore, our solutions for ( q , p ) (q,p) are ( 0 , 1 ) , ( 1 , 1 ) ( 18 , 7 ) ( 19 , 7 ) (0,1),\ (1,1)\ (-18,7)\ (19,7) Only ( 19 , 7 ) (19,7) gives two prime solutions, and so 2 N = 1 9 2 19 = 7 3 1 = 342 2N=19^2-19=7^3-1=342 The answer is 171 \boxed{171} .

Fiat Genzelogz
Aug 2, 2013

k 2 N 2 = p 3 k^2 - N^2 = p^3 --> ( k + N ) ( k N ) = p 3 (k+N)(k-N) = p^3

Which implies

k + N = p 2 , k N = p k+N = p^2 , k-N = p or k + N = p 3 , k N = p k+N = p^3, k-N = p

Writing the first case as a quadratic in terms of p and with constant N and using the quadratic formula, we see that 8 N + 1 8N + 1 at the very least must be a perfect square. Furthermore, this can only be true if N N is a triangle number. These numbers can be written \frac {(a)(a+1)/2}

Writing the second case in terms of p and N, we come to p 3 2 N 1 = 9 p^3 - 2N - 1 = 9 , implying 2 N + 1 2N + 1 is a perfect cube.

Substituting ( a ) ( a + 1 ) 2 \frac {(a)(a+1)}{2} into the second equation, we get a 2 + a + 1 = p 3 a^2 + a + 1 = p^3 /. Using the corollary of Ljunggren's theorem given, the only positive solution for a is 18. Thus N = ( 9 ) ( 17 ) = 171 N = (9)(17) = 171 .

Rodrigo Fischer
Aug 2, 2013

We have that p 3 + N 2 = x 2 p 3 = x 2 N 2 p 3 = ( x + N ) ( x N ) p^3 + N^2 = x^2 \rightarrow p^3 = x^2 - N^2 \rightarrow p^3 = (x + N)(x - N) . Since p is a prime, we have that ( x + N ) = p 2 (x + N) = p^2 and ( x N ) = p (x - N) = p or ( x + N ) = p 3 (x + N) = p'^3 and ( x N ) = 1 (x - N) = 1 for all values positive. Solving both systems, we lead to: 2 N = p ( p 1 ) 2N = p(p-1) and 2 N = p 3 1 2N = p'^3 - 1 . Lets call the first case as (1) and the second as (2). Making a system with (1) and (1) for the same N's we prove that there are no such primes that solve the system. Making a system with (2) and (2) we find the same thing. Combining (1) with (2) we get p 3 1 = p 2 p p 3 = p 2 p + 1 p'^3 - 1 = p^2 - p \rightarrow p'^3 = p^2 - p + 1 . We get to the Ljunggren equation but with the "p" with signal inverted. We look for the only solution with both x and y primes, wich is (-19,7). The real solution, since in our case the "p" is with the signal inverted, is (19,7). 2 N = 7 3 1 N = 171 2N = 7^3 - 1 \rightarrow N = 171 .

Nishant Sharma
Jul 30, 2013

Let p 3 + N 2 = K 2 p^{3} + N^{2}=\:K^{2} for some K Z + K\in \mathbb{Z^{+}}

p 3 = ( K N ) . ( K + N ) \implies\:p^{3}=\:(K - N).(K + N) --- ( i ) (i)

Since K K and N N are both positive integers K N < K + N \implies\:K - N<K + N so possible factorizations of ( i ) (i) are:

Case I: K N = p 1 K - N=\:p_{1} and K + N = p 1 2 K + N=\:p_{1}^{2} , , for some prime p 1 p_{1}

2 N = p 1 2 p 1 \implies\:2N=\:p_{1}^{2} - p_{1} --- ( i i ) (ii)

Case II: K N = 1 K - N=\:1 and K + N = p 2 3 K + N=\:p_{2}^{3} , , for some prime p 2 p_{2}

2 N = p 2 3 1 \implies\:2N=\:p_{2}^{3} - 1\;\;\; --- ( i i i ) (iii)

From ( i i ) (ii) and ( i i i ) (iii) we get

p 1 2 p 1 + 1 = p 2 3 p_{1}^{2} - p_{1} + 1=\:p_{2}^{3} --- ( i v ) (iv)

Since we know solutions to the equation x 2 + x + 1 = y 3 x^{2} + x + 1=\:y^{3} so we can get solutions to the equation x 2 x + 1 = y 3 x^{2} - x + 1=\:y^{3} by replacing x x with x -x . Using the Ljunggren Theorem solutions to ( i v ) (iv) are ( p 1 , p 2 ) = ( 0 , 1 ) , ( 1 , 1 ) , ( 18 , 7 ) , ( 19 , 7 ) . (p_{1},p_{2})=\:(0,1),(1,1),(-18,7),(19,7). Since a prime is always positive and greater than 1 1 so the only acceptable solution is ( p 1 , p 2 ) = ( 19 , 7 ) . (p_{1},p_{2})=\:(19,7).

Plugging in these values into their respective equations and verifying that K > 0 K>0 we can easily conclude that N = 19 × 9 = 171 . N=\:19\times9=\:\boxed{171}.

NOTE: We have used p 1 p_{1} and p 2 p_{2} only to say that there are two distinct values of p p satisfying given condition. It does not imply in any sense that p 1 = p 2 . p_{1}=\:p_{2}.

0 pending reports

×

Problem Loading...

Note Loading...

Set Loading...